LSAT and Law School Admissions Forum

Get expert LSAT preparation and law school admissions advice from PowerScore Test Preparation.

 Administrator
PowerScore Staff
  • PowerScore Staff
  • Posts: 8919
  • Joined: Feb 02, 2011
|
#40708
Complete Question Explanation
(The complete setup for this game can be found here: lsat/viewtopic.php?t=3641)

The correct answer choice is (D)

From the main game diagram, S cannot be edited second, fifth, or seventh. Only the fifth position appears among the answer choices to this Could Be True Except question, and thus answer choice (D) cannot occur and is correct.
 SamSachsDE
  • Posts: 12
  • Joined: Jun 28, 2018
|
#49927
Can you please provide a more detailed explanation on why S can not be in 5? The explanation refers to the diagram, and the diagram just says its a very difficult inference and involves the rules...
 Adam Tyson
PowerScore Staff
  • PowerScore Staff
  • Posts: 5153
  • Joined: Apr 14, 2011
|
#50039
Happy to oblige, SamSachsDE!

There are a few tools we could bring to bear on this question, starting with looking over our main diagram and any questions we have already answered to see where S CAN go, and then eliminating those answer choices. For example, we know from our answer to Question 17, the list question, that S can go third. That eliminates answer B. If we had, in the course of building our diagram, tried a hypothetical with S before Q, in order to test that rule, we would have discovered that S could go 1st, so answer A is eliminated. (And if we hadn't done that work in the setup phase, we probably would have done it in order to attack question 18, so we're covered).

None of the other question we've done to this point help much, so perhaps it's time to test the remaining answers?

Try S 4th, and here's what happens:

Q isn't 3rd, because then Q and S would be adjacent, so S cannot be before Q per that conditional rule. So Q is in the first three somewhere. Y is obviously after S, in the last three spaces somewhere. The big J-G-R sequence has to get broken up, with some of it before S and some after. And don't forget H, the random variable, which cannot be next to G or J.

Let's try putting JQG as the first three, and some combination of HRY as the last three, just making sure that R isn't next to S. A solution of JQGSYHR satisfies all the rules, and shows that S CAN be 4th. There are other solutions that would do the job, but we need not consider them because we only had to show that it works once in order to eliminate it.

On to answer D - what happens if S is 5th? Y must be after it. R cannot be next to it, but must be after J and G. Maybe we try putting R 7th, with Y between them? So far so good. Except now we have a problem - J, G, and H all still need to be placed, and they must be spaced away from each other, and there is no room to do that in the 4 spaces still left open. So R can't go last in this scenario - H or Y will have to go there. But now we have another problem - R cannot go 4th, next to S, and it can never go 3rd because that would force J and G to be adjacent at 1 and 2. With S 5th, there is no place to put R that will work with the remaining rules! Hence, S cannot go 5th, because there is no viable solution when it does.

Sometimes the only way to answer these questions is by brute force, just testing things out (after eliminating whatever you can, of course) until you find the right answer. Those are the questions that make an otherwise easy game harder, and a hard game horrible.

Play with it yourself a little by testing out each contender answer (like answer E, which we haven't discussed here, but which should work since D is the one that did not), and you'll get the hang of it. Follow ALL the rules, and complete the diagram. When you cannot, that's how you know you found that thing that cannot be true!

Have fun with it!

Get the most out of your LSAT Prep Plus subscription.

Analyze and track your performance with our Testing and Analytics Package.